What is System: Definition and 1000 Discussions

A system is a group of interacting or interrelated elements that act according to a set of rules to form a unified whole. A system, surrounded and influenced by its environment, is described by its boundaries, structure and purpose and expressed in its functioning. Systems are the subjects of study of systems theory.

View More On Wikipedia.org
  1. L

    Studying How to approach ME System Dynamics problems?

    In Summary: I have a test Friday in modeling mass-spring-pulley systems, and I'm struggling to tie everything together. We're using the textbook System Dynamics 3rd Edition by William Palm III, and while it does a great job describing the individual components of each system, it does a crappy...
  2. L

    Equation of Motion of a System of Springs (System Dynamics)

    I'm having trouble understanding the forces involved when defining the equation of motion for this particular problem. So... I've seen the answer for the eq. of motion worked two different ways. The first way involves combining the spring series into Keq = K1⋅K2/(K1+K2). The answer, plugging...
  3. Y

    I Thought experiment on buoyant forces on a system

    I'm a former physics student and I've been thinking about an interesting problem that eventually led me to the following thought experiment that I'm having trouble resolving. Imagine a two-compartment system, where one compartment is filled with He gas, and the other is filled with standard...
  4. M

    Minimum force before a system of two blocks will move

    Homework Statement [/B] ##m=3kg## ##M=2kg## Coefficient of static friction between m and M ##\mu_1 = 0.6## Coefficient of static friction between M and the ground ##\mu_2 = 0.8##. The rope is weightless, does not stretch, and the wheel is frictionless. What is the maximum force ##F## that...
  5. G

    Factoring Higher Order Polynomials: A Practical Method for Control System Design

    Homework Statement Determine the parameters of the PI controller such that two of the closed-loop poles of the transfer function Gclr(s) correspond to the poles of a second order LTI system with the following specifications: i) overshoot S% = 25%; and ii) settling time ts(5%) = 120 s. Homework...
  6. S

    Pulley System Part c: Homework Statement & Relevant Equation

    Homework Statement 2. Relevant equationThe Attempt at a Solution ...[/B] Part c) I'm confused as to how the actual pulley system works. If its balanced wouldn't it be stationary. How does it move? I thought that since its balanced the acceleration of the system is constant and therefore to...
  7. R

    Equation of an oscillating system without any starting values

    Homework Statement A mass m1 is located on a platform with mass M. The platfrom is located on springs with total constant k such that it can swing vertically in direction x. a) Write down the equations of motion assuming mass m1 will always be connected to the platform. Write it as x(t) b)...
  8. F

    Shock wave system of equations

    Homework Statement This is Rankine-Hugoniot conditions at a hydrodynamic shock front. Where P2=0 v2=0. The problem is attached. I need to solve a system of equations. I thought it would be relatively straight forward solving for the three unknowns but I'm struggling. I know it's possible to...
  9. Runei

    I Total Derivative of a Constrained System

    Hi all, I was working on a problem using Euler-Lagrange equations, and I started wondering about the total and partial derivatives. After some fiddling around in equations, I feel like I have confused myself a bit. I'm not a mathematician by training, so there must exist some terminology which...
  10. danielFiuza

    A Solution?: Quintic Equation from Physical System

    First time in this forum, so greetings to everyone! I am currently working with some physical models in the field of natural ventilation and I came across the following 5th order polynomial equation (quintic function): $X^{5}+ C X - C =0$ This is the steady state solution of a physical system...
  11. A

    Eigenfrequencies of a multiple DOF system

    I am confused with this concept. So if a system possesses multiple possible eigenfrequencies (and therefore modes), how does the system "know" which eigenfrequency will it want to vibrate on? Does that depend on the initial condition you give the system? Is there any mathematical relation...
  12. evinda

    MHB System of congruences, not relatively prime moduli

    Hello! (Wave) I want to solve the following system of congruences: $$x \equiv 13 \pmod{40} \\ x\equiv 5 \pmod{44} \\ x \equiv 38 \pmod{275}.$$I have thought the following: $$x \equiv 13 \pmod{40} \Leftrightarrow x \equiv 13 \pmod{2^3 \cdot 5}$$ $$x \equiv 5 \pmod{44} \Leftrightarrow x \equiv...
  13. S

    Determine the mass of an object using the angle in the system

    <Moderator's note: Moved from another forum and thus no template.> I have a project from my Mechanics class which consist on building the system on the picture. It has a spring, cable and a protractor. I have to do an approximation of the mass and weight of the object that is put in the...
  14. sams

    Scleronomic or Rheonomic Mechanical System?

    I would really appreciate if someone could advise me whether the system below is a scleronomic or a rheonomic mechanical system, or a mix of both. If we consider the first pendulum, the constraint is fixed which leads to a scleronomous case while the constraint of the second pendulum is not...
  15. gibberingmouther

    Momentum of a System and External Forces

    So Pearson is telling me that, basically, the ratio of internal to external forces and the briefness of the time interval is what determines whether the external forces on a system whose momentum we're studying will affect whether we can obtain a decent approximation of the momenta of the...
  16. J

    Wireless communications system

    Homework Statement A wireless communications system can achieve a theoretical maximum data rate of 200 Mbps using 16-QAM modulation in a 10 MHz channel and using a single spatial stream. What data rate could theoretically be achieved if the same system used 256-QAM and 8 spatial streams in a 20...
  17. S

    Force applied on the pulley of a system

    Homework Statement two blocks of masses 8kg and 4kg respectively are connected by a string as shown in firgure. calculate their accelerations if they are initially at rest on the floor. after a force of 100N is applied on the pulley in the upward direction (g=10 m/s). Homework Equations Σf=ma...
  18. S

    Acceleration and Tension in a Two-Block System on an Inclined Plane

    Homework Statement two blocks of mass m sit on a frictionless surface and are connected by a massless, unstrechable string that rolls on a massless, frictionless pulley as shown. the second mass sits on an inclined tipped at an angle θ relative to horizontal. at time t=0, both masses are...
  19. B

    Optics Problem with a Double Lens System

    Homework Statement An object is 25.0 cm to the left of a lens with a focal length of + 15 .0 cm. A second lens of focal length +15.0 cm is 30.0 cm to the right of the first lens. Find the location of the final image and draw it out. Homework Equations 1/f = 1/di + 1/do The Attempt at a...
  20. G

    System of multiple conductors: image method

    Homework Statement The following figure represent the traversal cut of a system with two cylindrical equal conductors of radius r0 length l at a distance d from one another and at the same distance h of a plane conductor (conductor zero). The dielectric that surrounds the conductors is the air...
  21. K

    Single transfer lag in an open-loop system

    Homework Statement FIGURE 3 shows an open loop system containing a distance velocity lag and a single transfer lag. Figure 3 shows - Xi Xo ---->----[Distance - velocity lag of...
  22. J

    Finding the acceleration of two masses on a pulley system

    Homework Statement Two blocks of the masses m1=7.40 kg and m2=m1/2 are connected via a massless pulley and massless string. The system is currently in equilibrium but is about to start sliding, if m2 would increase even by a bit. For the friction between the surface and m1 assume that µs=µk...
  23. cookiemnstr510510

    Calculating the capacitance of a system of two plates tilted at a small angle

    Homework Statement A capacitor is made up of a pair of square plates, each of side length L, that are very nearly parallel (such that θ is very small) and separated by a distance D. The plates make an angle, θ, with one another. The capacitor may be thought of as being comprised of many...
  24. M

    Problem involving a system of equations

    Homework Statement If an amount of $1000 is deposited in a savings account that pays 3.2% interest per year compounded monthly, the amount in the account after nmonths is given by: The amount in the account after 2 years (rounded to one decimal point) will be??
  25. G

    Partial capacitances of a system of conductors

    Homework Statement I have a small question about the following problem. The figure represents the cross-section of a three-conductor system comprising a communications coaxial cable of length l running parallel to a conducting wall (reference conductor). Determine the partial capacitance scheme...
  26. mktsgm

    Medical Sleeping immune system and effects

    What will happen if a pathogen invades us and our immune system is completely ignorant about it or sleeping? Let us suppose it fails to identify the intruders. Can the consequences be explained pathophysiologically by taking any germ as an example? I want to know how a pathogen causes...
  27. B

    What design of a drive system should I use?

    Hi there! I am currently making a design about drive systems used for micro-hydro scheme. I don't know what type of driving element is suitable to used for transmitting the power from a turbine (cross flow turbine) to a generator. I'm planning to use both chain and sprocket & V-Belt Pulley which...
  28. L

    Derivation of the Potential Energy of an Electric Charge System

    Hi, I learned about how PE = U=kq1q2/r is the electrical potential energy for the system. It is found by taking the integral of electrical force and dr from infinity to the point of location we are interested in. So that is the intregral(F*dr) from r=inf to r=ro. My question is that do I...
  29. jha192001

    Elongation of a spring in a two mass system

    The Question is posted as a photograph with a. little change. One of the mass in the photo is equal to '2m' rather than 'm'. Now the block won't stop consecutively. How would we approach the question?
  30. R

    A How do I pass a magnetic field to any coordinate system?

    Hello, For example, an electric field vector, such as the gradient of the potential, passes in the following way to any coordinate system:$$E = -\triangledown{}V = - \frac{{\partial V}}{{\partial x^i}}g^{ij}e_j$$ But what about a vector of a magnetic field? How would it be expressed in any...
  31. C

    B Treating a galaxy as a quantum system

    If a wave function could be assigned to a whole galaxy, would its mass spread along the wave? Could this account for the anomalies in our calculations for galactic spin?
  32. E

    Magnet system efficiency made from neodymium magnets

    Me and my friend want to make a strong as possible (made from permanent magnets) homemade metal separator for seeds. We are not good in the physics of such level, so we need consultation. We want to try neodymium magnets for our separator. We learned that magnetic flux density could be reached...
  33. S

    What is the total torque of that system?

    Homework Statement A load was put on the right side of a seesaw 0.6 meter from its center as shown, on the other side a force of 49N is applied perpendicular to the seesaw surface 1 meter from its center, the setup is tuned to an angle of 30 from the horizontal axis as shown. On the right...
  34. SidMe1984

    Calculating Total Weight of 10 Oscillators & 8 Quanta of Energy

    1. The problem statement Consider the case of 10 oscillators and eight quanta of energy. Determine the dominant configuration of energy for this system by identifying energy configurations and calculating the corresponding weights. What is the probability of observing the dominant...
  35. G

    Bloch equations for a 3-level system

    Homework Statement "Consider a system with three states, ##|1\rangle , |2\rangle ,|3\rangle ## with energies ##\hbar \omega_1 , \hbar \omega_2 , \hbar \omega_3 ##. the states are then separated by ##\hbar \omega_3 -\hbar \omega_1 = \hbar \omega_{13}## and ## \hbar \omega_3-\hbar \omega_2= \hbar...
  36. V

    Upgrading a conveyor system

    Hey guys, some insight to the following problem I have here would be helpful. I need to see if a conveyor is worth upgraded (i.e. see what the output Tonne/per hour is) from a belt 745mm wide, idler pulley 800mm wide... to a 800mm belt and 850mm wide idler pulley. Lets assume the length of the...
  37. M

    Why do extra moment equations not count toward a system of equations?

    I was just looking at indeterminate statics problems where you have a beam, three elastic wires (left, centre, and right that are holding it up), and some extra mass. (Just like example 5 in the notes below: http://fast10.vsb.cz/lausova/indeterm_all.pdf). I understand the method that was used to...
  38. DrClaude

    Two-level quantum system (from Sakurai)

    Homework Statement Sakurai, problem 1.11 A two-state system is characterized by the Hamiltonian $$ H = H_{11} | 1 \rangle \langle 1| + H_{22} | 2 \rangle \langle 2| + H_{12} \left[ | 1 \rangle \langle 2| + | 2 \rangle \langle 1| \right] $$ where ##H_{11}##, ##H_{22}##, and ##H_{12}## are real...
  39. M

    Write the state of the system after r photons leave the |nϕ> state

    Homework Statement (Note: this is the last part of a longer problem, so I hope I won't miss anything important) We have a laser with 2 energy levels, so the emitted photons will have the energy ##\omega##. For this given energy there are ##m## states in which the photon can be, denoted...
  40. Auto-Didact

    Lingusitics Language as a Dynamical System

    A few years ago I read two pretty groundbreaking linguistic papers from the 90s arguing that natural languages are networks which can be conceptualized from the perspective of nonlinear dynamical systems theory, with a lexicon being a state space and grammatical rules being attractors in that...
  41. U

    A Taylor expansion for a nonlinear system and Picard Iterations

    Hello guys I struggle since yesterday with the following problem I am reading the book "Elements of applied bifurcation theory" by Kuznetsov . At one point he has the following Taylor expansion of a nonlinear system with respect to x=0 where ##x\in \mathbb(R)^n## $$\dot{x} = f(x) = \Lambda x +...
  42. I

    "Innateness" of the "innate immune system"?

    Does every macrophage (and leucocyte in general) start from “scratch” as the organism grows in terms of "knowing" which pathogens to kill or does it somehow "know" to dismantle certain pathogens "genetically" once it is created? I guess, in other words, does it have to first get "exposed" to...
  43. Amitayas Banerjee

    What is the Lagrangian, equations of motion for this system?

    <<Moderator's note: Moved from a technical forum, no template.>> Description of the system: The masses m1 and m2 lie on a smooth surface. The masses are attached with a spring of non stretched length l0 and spring constant k. A constant force F is being applied to m2. My coordinates: Left of...
  44. Rahulx084

    Understanding the Concept of Heat Supply to a System: Explained in My Book

    My book states that Heat and work represent energy in transit. The state of the system undergoes a change after heat is supplied to it and before work is extracted from it because energy gets stored in it. My doubt is here that ,heat has been transferred to the system and it says heat is energy...
  45. R

    I Is a trip to explore the Alpha Centuri system actually feasible?

    Can the journey be completed within the lifetime of a human adult? Assuming we have got around little problems like having an adequate form of propulsion, shielding from radiation, and avoiding deadly collisions with milligram sized dust particles. Let's say our engines can provide constant...
  46. G

    A transmission system exercise

    Homework Statement We have a transmission system (plate-pinion), with a 38-tooth chainring and a 14-tooth sprocket. The distance between the crank (between the chainring and the pedal) is 170 mm and the pedal is overloaded with 60 kg and pedalled at a speed of 70 min-1. Homework Equations P =...
  47. sams

    I Do we consider a point in a coordinate system to be a scalar?

    Knowing that a scalar quantity doesn't change under rotation of a coordinate system. Do we consider a point in a Cartesian coordinate system (i.e. A (4,5)) a scalar quantity? If yes, why do the components of point A change under rotation of the coordinate system? According to my understanding...
  48. Incud2

    Find the resistive constant in a critically damped system

    Homework Statement This problem is taken from Problem 2.3, Introduction to Vibration and Waves, by H.J. Pain and P. Rankin: A critically mechanical system consisting of a pan hanging from a spring with a damping. What is the value of damping force r if a mass extends the spring by 10cm without...
  49. Wrichik Basu

    Delete system files from Android phone

    I have a Samsung Galaxy on7, running on Android 6.0.1. It has a lot of pre installed apps that I consider bloatware. I want to remove them completely from the phone. Recently, I tried removing these apps as per this video: But soon I found that it does not delete the system files, because...
  50. anemone

    MHB System of Equation Challenge (a+b)(b+c)=-1

    Given that a,\,b and c are real numbers that satisfy the system of equations below: (a+b)(b+c)=-1\\(a-b)^2+(a^2-b^2)^2=85\\(b-c)^2+(b^2-c^2)^2=75 Find (a-c)^2+(a^2-c^2)^2.
Back
Top